IASbaba Prelims 60 Days Plan, Rapid Revision Series (RaRe)
Archives
Hello Friends
The 60 Days Rapid Revision (RaRe) Series is IASbaba’s Flagship Initiative recommended by Toppers and loved by the aspirants’ community every year.
It is the most comprehensive program which will help you complete the syllabus, revise and practice tests on a daily basis. The Programme on a daily basis includes
Daily Prelims MCQs from Static (Monday – Saturday)
- Daily Static Quiz will cover all the topics of static subjects – Polity, History, Geography, Economics, Environment and Science and technology.
- 20 questions will be posted daily and these questions are framed from the topics mentioned in the schedule.
- It will ensure timely and streamlined revision of your static subjects.
Daily Current Affairs MCQs (Monday – Saturday)
- Daily 5 Current Affairs questions, based on sources like ‘The Hindu’, ‘Indian Express’ and ‘PIB’, would be published from Monday to Saturday according to the schedule.
Daily CSAT Quiz (Monday – Friday)
- CSAT has been an Achilles heel for many aspirants.
- Daily 5 CSAT Questions will be published.
Note – Daily Test of 20 static questions, 10 current affairs, and 5 CSAT questions. (35 Prelims Questions) in QUIZ FORMAT will be updated on a daily basis.
To Know More about 60 Days Rapid Revision (RaRe) Series – CLICK HERE
60 Day Rapid Revision (RaRe) Series Schedule – CLICK HERE
Important Note
- Comment your Scores in the Comment Section. This will keep you accountable, responsible and sincere in days to come.
- It will help us come out with the Cut-Off on a Daily Basis.
- Let us know if you enjoyed today’s test 🙂
- You can post your comments in the given format
- (1) Your Score
- (2) Matrix Meter
- (3) New Learning from the Test
Test-summary
0 of 35 questions completed
Questions:
- 1
- 2
- 3
- 4
- 5
- 6
- 7
- 8
- 9
- 10
- 11
- 12
- 13
- 14
- 15
- 16
- 17
- 18
- 19
- 20
- 21
- 22
- 23
- 24
- 25
- 26
- 27
- 28
- 29
- 30
- 31
- 32
- 33
- 34
- 35
Information
The following Test is based on the syllabus of 60 Days Plan-2023 for UPSC IAS Prelims 2022.
To view Solutions, follow these instructions:
- Click on – ‘Start Test’ button
- Solve Questions
- Click on ‘Test Summary’ button
- Click on ‘Finish Test’ button
- Now click on ‘View Questions’ button – here you will see solutions and links.
You have already completed the test before. Hence you can not start it again.
Test is loading...
You must sign in or sign up to start the test.
You have to finish following test, to start this test:
Results
0 of 35 questions answered correctly
Your time:
Time has elapsed
You have scored 0 points out of 0 points, (0)
Average score |
|
Your score |
|
Categories
- Not categorized 0%
Pos. | Name | Entered on | Points | Result |
---|---|---|---|---|
Table is loading | ||||
No data available | ||||
- 1
- 2
- 3
- 4
- 5
- 6
- 7
- 8
- 9
- 10
- 11
- 12
- 13
- 14
- 15
- 16
- 17
- 18
- 19
- 20
- 21
- 22
- 23
- 24
- 25
- 26
- 27
- 28
- 29
- 30
- 31
- 32
- 33
- 34
- 35
- Answered
- Review
-
Question 1 of 35
1. Question
Consider the following statements:
- A money bill cannot be introduced in Rajyasabha in any circumstances
- The Rajya Sabha cannot vote on the Demands for Grants in any circumstances
- The Rajya Sabha cannot pass a resolution for the discontinuation of the national emergency.
How many of the above statements are correct?
Correct
Solution (c)
Explanation:
The powers and status of the Rajya Sabha are unequal to that of the Lok Sabha in the following matters-
- A Money Bill can be introduced only in the Lok Sabha and not in the Rajya Sabha.
- Rajya Sabha cannot reject or amend a Money Bill. It should return the bill to the Lok Sabha within 14 days, either with recommendations or without recommendations.
- The Lok Sabha can either reject or accept all or any of the recommendations of the Rajya Sabha. In both the cases, the money bill is deemed to have been passed by the two Houses.
- A financial bill, not containing solely the matters of Article 110, also can be introduced only in the Lok Sabha and not in the Rajya Sabha. But, with regard to its passage, both the Houses have equal powers.
- The final power to decide whether a particular bill is a Money Bill or not is vested in the Speaker of the Lok Sabha.
- The Speaker of Lok Sabha presides over the joint sitting of both the Houses.
- The Lok Sabha with greater number wins the battle in a joint sitting except when the combined strength of the ruling party in both the Houses is less than that of the opposition parties.
- Rajya Sabha can only discuss the budget but cannot vote on the demands for grants (which is the exclusive privilege of the Lok Sabha).
- A resolution for the discontinuance of the national emergency can be passed only by the Lok Sabha and not by the Rajya Sabha.
- The Rajya Sabha cannot remove the council of ministers by passing a no-confidence motion. This is because the Council of ministers is collectively responsible only to the Lok Sabha. But, the Rajya Sabha can discuss and criticise the policies and activities of the government
Incorrect
Solution (c)
Explanation:
The powers and status of the Rajya Sabha are unequal to that of the Lok Sabha in the following matters-
- A Money Bill can be introduced only in the Lok Sabha and not in the Rajya Sabha.
- Rajya Sabha cannot reject or amend a Money Bill. It should return the bill to the Lok Sabha within 14 days, either with recommendations or without recommendations.
- The Lok Sabha can either reject or accept all or any of the recommendations of the Rajya Sabha. In both the cases, the money bill is deemed to have been passed by the two Houses.
- A financial bill, not containing solely the matters of Article 110, also can be introduced only in the Lok Sabha and not in the Rajya Sabha. But, with regard to its passage, both the Houses have equal powers.
- The final power to decide whether a particular bill is a Money Bill or not is vested in the Speaker of the Lok Sabha.
- The Speaker of Lok Sabha presides over the joint sitting of both the Houses.
- The Lok Sabha with greater number wins the battle in a joint sitting except when the combined strength of the ruling party in both the Houses is less than that of the opposition parties.
- Rajya Sabha can only discuss the budget but cannot vote on the demands for grants (which is the exclusive privilege of the Lok Sabha).
- A resolution for the discontinuance of the national emergency can be passed only by the Lok Sabha and not by the Rajya Sabha.
- The Rajya Sabha cannot remove the council of ministers by passing a no-confidence motion. This is because the Council of ministers is collectively responsible only to the Lok Sabha. But, the Rajya Sabha can discuss and criticise the policies and activities of the government
-
Question 2 of 35
2. Question
Which among the following is not provided for by the Constitution of India?
- Zero Hour
- Quorum
- Prorogation
- Calling Attention Motion
Choose the correct code:
Correct
Solution (b)
Statement 1 Statement 2 Statement 3 Statement 4 Correct Incorrect Incorrect Correct The Zero Hour is neither mentioned in the Constitution of India nor in the Rules of Procedure. Quorum is provided under Article 100 of the Constitution of India. Prorogation is provided under Article 85 of the Constitution of India. Calling Attention Motion is not mentioned in the Constitution of India. However, unlike the Zero Hour, it is mentioned in the Rules of Procedure. Note:
Zero Hour:
- Zero Hour is an India Specific Parliamentary device to discuss issues of urgent public importance.
- Without prior notice of 10 days, matters of national importance can be discussed during this hour by informing the Speaker at 10 AM on the day of the Parliamentary Session.
- Although Zero Hour is not specified in any Rule Book, presiding officers of both houses have worked to control it and make it more effective throughout the years.
- The Presiding officers in the Parliament regulate the zero by setting rules to streamline the process.
- In the Lok Sabha, Question Hour comes first, followed by Zero Hour, however in the Rajya Sabha, it is the other way around.
- The Difference between Zero Hour and Question Hour is the Ministers are not bound to reply in the Zero Hour.
Quorum:
- The Quorum refers to the number of members that must be present for a house meeting to take place.
- The Constitution established a quorum of one-tenth of the total membership for both the Lok Sabha and the Rajya Sabha.
- Thus, at least 55 members must be present to hold a Lok Sabha session, while at least 25 members must be present to conduct a Rajya Sabha sitting.
- For both the Lok Sabha and the Rajya Sabha, a quorum of one-tenth of the total number of members is required to convene a meeting.
- If a quorum is not present, the Speaker must adjourn the House or postpone the meeting until a quorum is present.
- For both the Lok Sabha and the Rajya Sabha, the Constitution has established a quorum of one-tenth of the total membership. Individual members have a six-year commitment.
- Whenever there are casual vacancies, they are replaced through bye-elections for the remainder of the term.
Prorogation:
- Prorogation means the termination of a session of the House by an order made by the President under article 85(2)(a) of the Constitution. Prorogation terminates both the sitting and session of the House.
- Usually, within a few days after the House is adjourned sine die by the presiding officer, the President issues a notification for the prorogation of the session. However, the President can also prorogue the House while in session.
- All pending notices (other than those for introducing bills) lapse on prorogation and fresh notices have to be given for the next session.
Calling Attention Motion:
- A member may, with the Speaker’s approval, move a motion to bring the attention of a Minister to any topic of urgent public interest.
- The Minister may make a brief statement or request additional time to make a statement at a later time.
- A single member can only make two call-attention motions in a single session.
- It is an Indian innovation similar to “zero hours” but finds a place in the rules of the procedure since 1952.
- It can be introduced in either of the houses of the Parliament.
Incorrect
Solution (b)
Statement 1 Statement 2 Statement 3 Statement 4 Correct Incorrect Incorrect Correct The Zero Hour is neither mentioned in the Constitution of India nor in the Rules of Procedure. Quorum is provided under Article 100 of the Constitution of India. Prorogation is provided under Article 85 of the Constitution of India. Calling Attention Motion is not mentioned in the Constitution of India. However, unlike the Zero Hour, it is mentioned in the Rules of Procedure. Note:
Zero Hour:
- Zero Hour is an India Specific Parliamentary device to discuss issues of urgent public importance.
- Without prior notice of 10 days, matters of national importance can be discussed during this hour by informing the Speaker at 10 AM on the day of the Parliamentary Session.
- Although Zero Hour is not specified in any Rule Book, presiding officers of both houses have worked to control it and make it more effective throughout the years.
- The Presiding officers in the Parliament regulate the zero by setting rules to streamline the process.
- In the Lok Sabha, Question Hour comes first, followed by Zero Hour, however in the Rajya Sabha, it is the other way around.
- The Difference between Zero Hour and Question Hour is the Ministers are not bound to reply in the Zero Hour.
Quorum:
- The Quorum refers to the number of members that must be present for a house meeting to take place.
- The Constitution established a quorum of one-tenth of the total membership for both the Lok Sabha and the Rajya Sabha.
- Thus, at least 55 members must be present to hold a Lok Sabha session, while at least 25 members must be present to conduct a Rajya Sabha sitting.
- For both the Lok Sabha and the Rajya Sabha, a quorum of one-tenth of the total number of members is required to convene a meeting.
- If a quorum is not present, the Speaker must adjourn the House or postpone the meeting until a quorum is present.
- For both the Lok Sabha and the Rajya Sabha, the Constitution has established a quorum of one-tenth of the total membership. Individual members have a six-year commitment.
- Whenever there are casual vacancies, they are replaced through bye-elections for the remainder of the term.
Prorogation:
- Prorogation means the termination of a session of the House by an order made by the President under article 85(2)(a) of the Constitution. Prorogation terminates both the sitting and session of the House.
- Usually, within a few days after the House is adjourned sine die by the presiding officer, the President issues a notification for the prorogation of the session. However, the President can also prorogue the House while in session.
- All pending notices (other than those for introducing bills) lapse on prorogation and fresh notices have to be given for the next session.
Calling Attention Motion:
- A member may, with the Speaker’s approval, move a motion to bring the attention of a Minister to any topic of urgent public interest.
- The Minister may make a brief statement or request additional time to make a statement at a later time.
- A single member can only make two call-attention motions in a single session.
- It is an Indian innovation similar to “zero hours” but finds a place in the rules of the procedure since 1952.
- It can be introduced in either of the houses of the Parliament.
-
Question 3 of 35
3. Question
Consider the following statements regarding Motions:
- Substantive Motion is a self-contained independent proposal dealing with a very important matter.
- Substitute Motion is a motion that, by itself, has no meaning and cannot state the decision of the House without reference to the original motion or proceedings of the House.
- Subsidiary Motion is a motion that is moved in substitution of an original motion and proposes an alternative to it.
How many of the above statements are correct?
Correct
Solution (a)
Statement 1 Statement 2 Statement 3 Correct Incorrect Incorrect Substantive Motion is a self-contained independent proposal dealing with a very important matter like impeachment of the President or removal of Chief Election Commissioner. Substitute Motion is a motion that is moved in substitution of an original motion and proposes an alternative to it. Subsidiary Motion is a motion that, by itself, has no meaning and cannot state the decision of the House without reference to the original motion or proceedings of the House. Note:
Different Categories of Motions:
All motions received in the Lok Sabha Secretariat under the rules shall be classified under the following categories:
- Substantive Motions:
A substantive motion is a self-contained independent proposal submitted for the approval of the House and drafted in such a way as to be capable of expressing a decision of the House, e.g., all resolutions are substantive motions.
- Substitute Motions:
Motions moved in substitution of the original motion for taking into consideration a policy or situation or statement or any other matter are called substitute motions
- Subsidiary Motions:
They depend upon or relate to other motions or follow upon some proceedings in the House. They by themselves have no meaning and are not capable of stating the decision of the House without reference to the original motion or proceedings of the House.
Subsidiary Motions are further divided into three classes-
- Ancillary Motions:
They are motions that are recognized by the practice of the House as the regular way of proceeding with various kinds of business. The following are the examples of ancillary motions, namely:
- That the Bill be taken into consideration
- That the Bill be passed
- Superseding Motions:
They are motions which, though independent in form, are moved in the course of the debate on another question and seek to supersede that question. In that class all the dilatory motions fall. The following motions are superseding motions in relation to the motion for taking into consideration a Bill:
- That the Bill be re-committed to a Select Committee
- That the Bill be re-committed to a Joint Committee of the Houses
- Amendments:
They are subsidiary motions that interpose a new process of question and decision between the main question and its decision. Amendments may be to the clause of a Bill, to a resolution or to a motion, or to an amendment to a clause of a Bill, resolution or motion.
Incorrect
Solution (a)
Statement 1 Statement 2 Statement 3 Correct Incorrect Incorrect Substantive Motion is a self-contained independent proposal dealing with a very important matter like impeachment of the President or removal of Chief Election Commissioner. Substitute Motion is a motion that is moved in substitution of an original motion and proposes an alternative to it. Subsidiary Motion is a motion that, by itself, has no meaning and cannot state the decision of the House without reference to the original motion or proceedings of the House. Note:
Different Categories of Motions:
All motions received in the Lok Sabha Secretariat under the rules shall be classified under the following categories:
- Substantive Motions:
A substantive motion is a self-contained independent proposal submitted for the approval of the House and drafted in such a way as to be capable of expressing a decision of the House, e.g., all resolutions are substantive motions.
- Substitute Motions:
Motions moved in substitution of the original motion for taking into consideration a policy or situation or statement or any other matter are called substitute motions
- Subsidiary Motions:
They depend upon or relate to other motions or follow upon some proceedings in the House. They by themselves have no meaning and are not capable of stating the decision of the House without reference to the original motion or proceedings of the House.
Subsidiary Motions are further divided into three classes-
- Ancillary Motions:
They are motions that are recognized by the practice of the House as the regular way of proceeding with various kinds of business. The following are the examples of ancillary motions, namely:
- That the Bill be taken into consideration
- That the Bill be passed
- Superseding Motions:
They are motions which, though independent in form, are moved in the course of the debate on another question and seek to supersede that question. In that class all the dilatory motions fall. The following motions are superseding motions in relation to the motion for taking into consideration a Bill:
- That the Bill be re-committed to a Select Committee
- That the Bill be re-committed to a Joint Committee of the Houses
- Amendments:
They are subsidiary motions that interpose a new process of question and decision between the main question and its decision. Amendments may be to the clause of a Bill, to a resolution or to a motion, or to an amendment to a clause of a Bill, resolution or motion.
-
Question 4 of 35
4. Question
Consider the following statements related to the Parliament
- Parliament must be convened by the President at least once in every six months.
- The duration of the session is decided by the President.
- The time allocated for discussing Bills is determined by the Business Advisory Committee, consisting of members from both the ruling and opposition parties.
How many of the above statements are correct?
Correct
Solution (b)
Statement 1 Statement 2 Statement 3 Correct Incorrect Correct Parliament must be convened by the President at least once in every six months. The power to convene a session of Parliament rests with the government. The decision is taken by the Cabinet Committee on Parliamentary Affairs, which currently comprises nine ministers, including those for Defence, Home, Finance, and Law. The decision of the Committee is formalised by the President, in whose name MPs are summoned to meet for a session The time allocated for discussing Bills is determined by the Business Advisory Committee, consisting of members from both the ruling and opposition parties. Note:
- India does not have a fixed parliamentary calendar.
- The session of parliament in India is convened by the Government.
- The President summons each House of the Parliament from time to time.
- The gap between two sessions of the Parliament cannot exceed 6 months, which means the Parliament meets at least two times in one year.
- By convention (i.e., not provided by the Constitution), Parliament meets for three sessions in a year.
Sessions of Parliament:
- A session of the Indian Parliament is the period during which a House meets almost every day uninterruptedly to manage the business.
- There are typically three sessions in a year. A session contains many meetings.
- The process of calling all members of the Parliament to meet is called Summoning of Parliament.
- It is the President who summons Parliament.
- In general, the sessions are as follows:
- Budget session (February to May)
- Monsoon session (July to September)
- Winter session (November to December)
Budget Session:
- The budget session was usually held from February to May every year.
- It is considered to be a highly crucial session of the Parliament.
- The Budget is usually presented on the last working day of the month of February.
- Here, the members discuss the various provisions of the budget and matters concerning taxation, after the Finance Minister presents the budget.
- The budget session is generally split into two periods with a gap of one month between them.
- This session every year starts with the President’s Address to both Houses.
- Go through the details of Union Budget 2022 on the given link.
Monsoon Session:
- The monsoon session is held in July to September every year.
- This is after a break of two months after the budget session.
- In this session, matters of public interest are discussed.
Winter Session:
- The winter session of Parliament is held in mid-November to mid-December every year.
- It is the shortest session of all.
- It takes up the matters that could not be considered upon earlier and makes up for the absence of legislative business during the second session of the Parliament.
Incorrect
Solution (b)
Statement 1 Statement 2 Statement 3 Correct Incorrect Correct Parliament must be convened by the President at least once in every six months. The power to convene a session of Parliament rests with the government. The decision is taken by the Cabinet Committee on Parliamentary Affairs, which currently comprises nine ministers, including those for Defence, Home, Finance, and Law. The decision of the Committee is formalised by the President, in whose name MPs are summoned to meet for a session The time allocated for discussing Bills is determined by the Business Advisory Committee, consisting of members from both the ruling and opposition parties. Note:
- India does not have a fixed parliamentary calendar.
- The session of parliament in India is convened by the Government.
- The President summons each House of the Parliament from time to time.
- The gap between two sessions of the Parliament cannot exceed 6 months, which means the Parliament meets at least two times in one year.
- By convention (i.e., not provided by the Constitution), Parliament meets for three sessions in a year.
Sessions of Parliament:
- A session of the Indian Parliament is the period during which a House meets almost every day uninterruptedly to manage the business.
- There are typically three sessions in a year. A session contains many meetings.
- The process of calling all members of the Parliament to meet is called Summoning of Parliament.
- It is the President who summons Parliament.
- In general, the sessions are as follows:
- Budget session (February to May)
- Monsoon session (July to September)
- Winter session (November to December)
Budget Session:
- The budget session was usually held from February to May every year.
- It is considered to be a highly crucial session of the Parliament.
- The Budget is usually presented on the last working day of the month of February.
- Here, the members discuss the various provisions of the budget and matters concerning taxation, after the Finance Minister presents the budget.
- The budget session is generally split into two periods with a gap of one month between them.
- This session every year starts with the President’s Address to both Houses.
- Go through the details of Union Budget 2022 on the given link.
Monsoon Session:
- The monsoon session is held in July to September every year.
- This is after a break of two months after the budget session.
- In this session, matters of public interest are discussed.
Winter Session:
- The winter session of Parliament is held in mid-November to mid-December every year.
- It is the shortest session of all.
- It takes up the matters that could not be considered upon earlier and makes up for the absence of legislative business during the second session of the Parliament.
-
Question 5 of 35
5. Question
Consider the following statements regarding Speaker Pro-Tem?
- He presides over the Lame-Duck session of the Parliament.
- He is appointed by the President of India.
- According to the Constitution, the senior most member of the Lok Sabha is appointed as the Speaker Pro-Tem.
How many of the above statements are correct?
Correct
Solution (a)
Statement 1 Statement 2 Statement 3 Incorrect Correct Incorrect Speaker pro-tem presides over the first sitting of the newly-elected Lok Sabha not over Lame-Duck Session. He is appointed by the President of India. Usually, the senior most member is selected as Speaker Pro-Tem. The President himself administers oath to the Speaker Pro-Tem. It is not a constitutional provision.
Speaker pro-term:
- As provided by the Constitution, the Speaker of the last Lok Sabha vacates his office immediately before the first meeting of the newly-elected Lok Sabha.
- Therefore, the President appoints a member of the Lok Sabha as the Speaker Pro-Tem.
- Usually, the senior most member is selected for this. The President himself administers oath to the Speaker Pro-Tem.
- The Speaker Pro-Tem has all the powers of the Speaker. He presides over the first sitting of the newly-elected Lok Sabha.
- When the new Speaker is elected by the House, the office of the Speaker Pro-Tem ceases to exist. Hence, this office is a temporary office, existing for a few days.
Incorrect
Solution (a)
Statement 1 Statement 2 Statement 3 Incorrect Correct Incorrect Speaker pro-tem presides over the first sitting of the newly-elected Lok Sabha not over Lame-Duck Session. He is appointed by the President of India. Usually, the senior most member is selected as Speaker Pro-Tem. The President himself administers oath to the Speaker Pro-Tem. It is not a constitutional provision.
Speaker pro-term:
- As provided by the Constitution, the Speaker of the last Lok Sabha vacates his office immediately before the first meeting of the newly-elected Lok Sabha.
- Therefore, the President appoints a member of the Lok Sabha as the Speaker Pro-Tem.
- Usually, the senior most member is selected for this. The President himself administers oath to the Speaker Pro-Tem.
- The Speaker Pro-Tem has all the powers of the Speaker. He presides over the first sitting of the newly-elected Lok Sabha.
- When the new Speaker is elected by the House, the office of the Speaker Pro-Tem ceases to exist. Hence, this office is a temporary office, existing for a few days.
-
Question 6 of 35
6. Question
Which of the following may be recognized as the leader of the opposition in the Parliament?
Correct
Solution (a)
Leader of Opposition:
- In each House of Parliament, there is the ‘Leader of the Opposition’. The leader of the largest Opposition party having not less than one-tenth seats of the total strength of the House is recognised as the leader of the Opposition in that House.
- In a parliamentary system of government, the leader of the opposition has a significant role to play.
- His main functions are to provide a constructive criticism of the policies of the government and to provide an alternative government.
- Therefore, the leader of Opposition in the Lok Sabha and the Rajya Sabha were accorded statutory recognition in 1977.
- They are also entitled to the salary, allowances and other facilities equivalent to that of a cabinet minister.
- It was in 1969 that an official leader of the opposition was recognised for the first time.
- The same functionary in USA is known as the ‘minority leader’.
Incorrect
Solution (a)
Leader of Opposition:
- In each House of Parliament, there is the ‘Leader of the Opposition’. The leader of the largest Opposition party having not less than one-tenth seats of the total strength of the House is recognised as the leader of the Opposition in that House.
- In a parliamentary system of government, the leader of the opposition has a significant role to play.
- His main functions are to provide a constructive criticism of the policies of the government and to provide an alternative government.
- Therefore, the leader of Opposition in the Lok Sabha and the Rajya Sabha were accorded statutory recognition in 1977.
- They are also entitled to the salary, allowances and other facilities equivalent to that of a cabinet minister.
- It was in 1969 that an official leader of the opposition was recognised for the first time.
- The same functionary in USA is known as the ‘minority leader’.
-
Question 7 of 35
7. Question
Consider the following statements regarding Lame-duck session:
- It is conducted after election of new members but before they are installed.
- It refers to a session in which members participated for last time because of failure to re-election in Lok Sabha.
- Lame-duck session requires minimum number of members whose presence is essential to transact the business of the House.
How many of the above statements are correct?
Correct
Solution (b)
Statement 1 Statement 2 Statement 3 Correct Correct Incorrect It is conducted after election of new members but before they are installed. It refers to a session in which members participated for last time because of failure to re-election in Lok Sabha. It does not require minimum number of members whose presence is essential to transact the business of the House. Lame-duck session:
- It is conducted after election of new members but before they are installed.
- It refers to a session in which members participated for last time because of failure to re-election in Lok Sabha.
- Lame-duck session do not usually occur in countries under a parliamentary form of government, whether the Westminster system or the other modules
- Those members of the existing Lok Sabha who could not get re-elected to the new Lok Sabha are called lame-ducks.
Incorrect
Solution (b)
Statement 1 Statement 2 Statement 3 Correct Correct Incorrect It is conducted after election of new members but before they are installed. It refers to a session in which members participated for last time because of failure to re-election in Lok Sabha. It does not require minimum number of members whose presence is essential to transact the business of the House. Lame-duck session:
- It is conducted after election of new members but before they are installed.
- It refers to a session in which members participated for last time because of failure to re-election in Lok Sabha.
- Lame-duck session do not usually occur in countries under a parliamentary form of government, whether the Westminster system or the other modules
- Those members of the existing Lok Sabha who could not get re-elected to the new Lok Sabha are called lame-ducks.
-
Question 8 of 35
8. Question
Consider the following statements about the Anti-Defection Law:
- There is a fixed timeline before which the Speaker needs to decide over the disqualification of the members.
- A member, who has been disqualified under the Anti-defection Law, can be appointed to a remunerative political post.
- The disqualification of a member under the Anti-Defection Law bars the member for being re-elected as a member of the House.
- A member who has resigned from the House cannot be disqualified under the Anti-Defection Law.
How many of the above statements are incorrect?
Correct
Solution (d)
Statement 1 Statement 2 Statement 3 Statement 4 Incorrect Incorrect Incorrect Incorrect There is no fixed timeline in which the Speaker has to decide on the disqualification of the members. Article 361 B: A member of a House, belonging to any political party, who is disqualified for being a member of the House under paragraph 2 of the Tenth Schedule, shall also be disqualified to hold any remunerative political post for the duration of the period commencing from the date of his disqualification, till the date on which the term of his office as such member would expire, or till the date on which he contests an election to a House and is declared elected, whichever is earlier. Disqualification from being a member of the House is not allowed under the Anti-Defection Law. The defected members can be re-elected to the House. In the Shrimanth Balasaheb Patil vs the Speaker of Karnataka Assembly Case, the Supreme Court observed that the Speaker can disqualify a member of the House, who has resigned from the House with the intention of defecting from whips directions. Incorrect
Solution (d)
Statement 1 Statement 2 Statement 3 Statement 4 Incorrect Incorrect Incorrect Incorrect There is no fixed timeline in which the Speaker has to decide on the disqualification of the members. Article 361 B: A member of a House, belonging to any political party, who is disqualified for being a member of the House under paragraph 2 of the Tenth Schedule, shall also be disqualified to hold any remunerative political post for the duration of the period commencing from the date of his disqualification, till the date on which the term of his office as such member would expire, or till the date on which he contests an election to a House and is declared elected, whichever is earlier. Disqualification from being a member of the House is not allowed under the Anti-Defection Law. The defected members can be re-elected to the House. In the Shrimanth Balasaheb Patil vs the Speaker of Karnataka Assembly Case, the Supreme Court observed that the Speaker can disqualify a member of the House, who has resigned from the House with the intention of defecting from whips directions. -
Question 9 of 35
9. Question
A member of parliament who is not a minister wishes to introduce a bill on an issue of national interest in the Lok Sabha. Which of the following criteria must be fulfilled by the member to introduce such a bill?
- The member should belong to the opposition party.
- He must give a one month notice to the House for the permission to introduce the bill.
- As per the convention, the bill can be introduced only on Friday.
- He must get the president approval to introduce the bill.
How many of the above statements are correct?
Correct
Solution (b)
Statement 1 Statement 2 Statement 3 Statement 4 Incorrect Correct Correct Incorrect Non minister MP can introduce private bill where that MP can be from any party and not necessarily from opposition party. Notice period for introduction of the private bill before the house should be a minimum of one month notice to examine its compliance with rules of house and constitutional provisions. Private bill is introduced only on Friday, a popular convention of Indian parliamentary form of government, while government bill can be introduced on any day. The nature of bill, whether ordinary or money bill, decides the need by the MP to get prior recommendation of the president. The type of bill, whether private or government bill, doesn’t decide the need to get president approval. Note:
PRIVATE MEMBER’S BILL:
- Introduction of a legislative bill done by a MP who is not a minister is referred to as private member bill while bills introduced by ministers are known as government bills. Individual members may introduce private bills to draw attention to issues requiring intervention of parliament.
- Notice period for introduction of the private bill before the house should be a minimum of one month notice to examine its compliance with rules of house and constitutional provisions.
- Admission of the bill is decided by the speaker in the case of Lok Sabha and chairman in the case of Rajya Sabha. It is generally admitted to be introduced only on Friday, a popular convention of Indian parliamentary form of government, while government bill can be introduced on any day.
- Till date only 14 private bills have been passed where none of them have been passed in the parliament post 1970.
GOVERNMENT BILL PRIVATE BILL It is introduced in the parliament by a minister. It is introduced in the parliament by non-minister MP. It reflects the policy of the government or the ruling majority party. It reflects the political mood of an important public matter. It has greater chance to be passed as the minister will be with the majority support of the ruling party. It is less likely to be passed by the parliament. Its introduction in the house requires 7 days’ notice period only. Introduction of the private bill before the house should be a minimum of one month notice to examine its compliance with rules of house and constitutional provisions. It is drafted with the assistance from ministry of law and attorney general. Drafting of the bill is the sole responsibility and concern of the non-minister MP. Incorrect
Solution (b)
Statement 1 Statement 2 Statement 3 Statement 4 Incorrect Correct Correct Incorrect Non minister MP can introduce private bill where that MP can be from any party and not necessarily from opposition party. Notice period for introduction of the private bill before the house should be a minimum of one month notice to examine its compliance with rules of house and constitutional provisions. Private bill is introduced only on Friday, a popular convention of Indian parliamentary form of government, while government bill can be introduced on any day. The nature of bill, whether ordinary or money bill, decides the need by the MP to get prior recommendation of the president. The type of bill, whether private or government bill, doesn’t decide the need to get president approval. Note:
PRIVATE MEMBER’S BILL:
- Introduction of a legislative bill done by a MP who is not a minister is referred to as private member bill while bills introduced by ministers are known as government bills. Individual members may introduce private bills to draw attention to issues requiring intervention of parliament.
- Notice period for introduction of the private bill before the house should be a minimum of one month notice to examine its compliance with rules of house and constitutional provisions.
- Admission of the bill is decided by the speaker in the case of Lok Sabha and chairman in the case of Rajya Sabha. It is generally admitted to be introduced only on Friday, a popular convention of Indian parliamentary form of government, while government bill can be introduced on any day.
- Till date only 14 private bills have been passed where none of them have been passed in the parliament post 1970.
GOVERNMENT BILL PRIVATE BILL It is introduced in the parliament by a minister. It is introduced in the parliament by non-minister MP. It reflects the policy of the government or the ruling majority party. It reflects the political mood of an important public matter. It has greater chance to be passed as the minister will be with the majority support of the ruling party. It is less likely to be passed by the parliament. Its introduction in the house requires 7 days’ notice period only. Introduction of the private bill before the house should be a minimum of one month notice to examine its compliance with rules of house and constitutional provisions. It is drafted with the assistance from ministry of law and attorney general. Drafting of the bill is the sole responsibility and concern of the non-minister MP. -
Question 10 of 35
10. Question
Which of the following options are available to the Lok Sabha to prevent arbitrary functioning of the Union Executive?
- Rejection of a money bill.
- Passing an adjournment motion.
- Non-approval of Motion of Thanks.
- Passing a cut motion.
Select the correct answer using the code given below:
Correct
Solution (d)
- The council of ministers can be removed from office by the Lok Sabha by passing a no-confidence motion.
- The Lok Sabha can also express lack of confidence in the government in the following ways:
- By not passing a motion of thanks on the President’s inaugural address.
- By rejecting a money bill.
- By passing a censure motion or an adjournment motion.
- By defeating the government on a vital issue.
- By passing a cut motion.
Note:
No-Confidence Motion:
- Article 75 of the Constitution says that the council of ministers shall be collectively responsible to the Lok Sabha. It means that the ministry stays in office so long as it enjoys confidence of the majority of the members of the Lok Sabha.
- In other words, the Lok Sabha can remove the ministry from office by passing a no-confidence motion. The motion needs the support of 50 members to be admitted.
Motion of Thanks:
- The first session after each general election and the first session of every fiscal year is addressed by the president.
- In this address, the president outlines the policies and programmes of the government in the preceding year and ensuing year. This address of the president, which corresponds to the ‘speech from the Throne in Britain’, is discussed in both the Houses of Parliament on a motion called the ‘Motion of Thanks’.
- At the end of the discussion, the motion is put to vote. This motion must be passed in the House. Otherwise, it amounts to the defeat of the government.
Censure Motion:
- It is moved for censuring the council of ministers for specific policies and actions.
- If it is passed in the Lok Sabha, the council of ministers need not resign from the office.
Adjournment Motion:
- It is introduced in the Parliament to draw attention of the House to a definite matter of urgent public importance, and needs the support of 50 members to be admitted.
- It involves an element of censure against the government and hence Rajya Sabha is not permitted to make use of this device.
Incorrect
Solution (d)
- The council of ministers can be removed from office by the Lok Sabha by passing a no-confidence motion.
- The Lok Sabha can also express lack of confidence in the government in the following ways:
- By not passing a motion of thanks on the President’s inaugural address.
- By rejecting a money bill.
- By passing a censure motion or an adjournment motion.
- By defeating the government on a vital issue.
- By passing a cut motion.
Note:
No-Confidence Motion:
- Article 75 of the Constitution says that the council of ministers shall be collectively responsible to the Lok Sabha. It means that the ministry stays in office so long as it enjoys confidence of the majority of the members of the Lok Sabha.
- In other words, the Lok Sabha can remove the ministry from office by passing a no-confidence motion. The motion needs the support of 50 members to be admitted.
Motion of Thanks:
- The first session after each general election and the first session of every fiscal year is addressed by the president.
- In this address, the president outlines the policies and programmes of the government in the preceding year and ensuing year. This address of the president, which corresponds to the ‘speech from the Throne in Britain’, is discussed in both the Houses of Parliament on a motion called the ‘Motion of Thanks’.
- At the end of the discussion, the motion is put to vote. This motion must be passed in the House. Otherwise, it amounts to the defeat of the government.
Censure Motion:
- It is moved for censuring the council of ministers for specific policies and actions.
- If it is passed in the Lok Sabha, the council of ministers need not resign from the office.
Adjournment Motion:
- It is introduced in the Parliament to draw attention of the House to a definite matter of urgent public importance, and needs the support of 50 members to be admitted.
- It involves an element of censure against the government and hence Rajya Sabha is not permitted to make use of this device.
-
Question 11 of 35
11. Question
With reference to the Parliamentary Committees, consider the following statements about the Estimates Committee:
- The members of Lok Sabha, as well as Rajya Sabha, find representation in the committee.
- The members are elected according to the principle of proportional representation.
- The demand for grants cannot be voted by the Lok Sabha without the report from the Estimates Committee.
How many of the above statements are correct?
Correct
Solution (a)
Statement 1 Statement 2 Statement 3 Incorrect Correct Incorrect Estimates committee compromises of 30 members all from Lok Sabha because only they can scrutinize the demand of grants of various ministries as a postmortem function in the late period of time. Election procedure followed is proportional representation system by single transferable vote to choose members among themselves so that all sections of the house are represented. Estimates committee is also known as postmortem committee as it reviews the demand of grants after they have been sanctioned to create deterrence among ministers for upholding policy standard. Note:
FINANCIAL COMMITTEES PUBLIC ACCOUNTS COMMITTEE ESTIMATES COMMITTEE COMMITTEE ON PUBLIC UNDERTAKING Established in 1921 with inference from Government of India act 1919. Instituted in 1951 based on John Mathai committee recommendations. Established in 1964 under Krishna Menon committee recommendations which made public sector units accountable to the parliament. Comprises of 15 Lok Sabha and 7 Rajya Sabha members. Compromises of 30 members all from Lok Sabha. It has 22 members with 15 from Lok Sabha and 7 from Rajya Sabha. Election procedure is proportional representation system by single transferable vote. Election procedure is proportional representation system by single transferable vote. Election procedure is proportional representation system by single transferable vote. Ministers cannot be the members of these committees as the role and function of these three committees are to check the functioning, activities and accountability of the government. Chairman of the public accounts committee, estimates committee and committee on public undertaking will be appointed by speaker of the Lok Sabha. The tenure of all these three committees is confined to one year and are reconstituted at the start of the year. Incorrect
Solution (a)
Statement 1 Statement 2 Statement 3 Incorrect Correct Incorrect Estimates committee compromises of 30 members all from Lok Sabha because only they can scrutinize the demand of grants of various ministries as a postmortem function in the late period of time. Election procedure followed is proportional representation system by single transferable vote to choose members among themselves so that all sections of the house are represented. Estimates committee is also known as postmortem committee as it reviews the demand of grants after they have been sanctioned to create deterrence among ministers for upholding policy standard. Note:
FINANCIAL COMMITTEES PUBLIC ACCOUNTS COMMITTEE ESTIMATES COMMITTEE COMMITTEE ON PUBLIC UNDERTAKING Established in 1921 with inference from Government of India act 1919. Instituted in 1951 based on John Mathai committee recommendations. Established in 1964 under Krishna Menon committee recommendations which made public sector units accountable to the parliament. Comprises of 15 Lok Sabha and 7 Rajya Sabha members. Compromises of 30 members all from Lok Sabha. It has 22 members with 15 from Lok Sabha and 7 from Rajya Sabha. Election procedure is proportional representation system by single transferable vote. Election procedure is proportional representation system by single transferable vote. Election procedure is proportional representation system by single transferable vote. Ministers cannot be the members of these committees as the role and function of these three committees are to check the functioning, activities and accountability of the government. Chairman of the public accounts committee, estimates committee and committee on public undertaking will be appointed by speaker of the Lok Sabha. The tenure of all these three committees is confined to one year and are reconstituted at the start of the year. -
Question 12 of 35
12. Question
Consider the following statements regarding Statutory Grants:
- Statutory Grants are granted under Article 282.
- These grants are charged on the Consolidated Fund of India every year.
- The statutory grants are given to the states on the recommendation of the Finance Commission.
How many of the above statements are correct?
Correct
Solution (b)
Statement 1 Statement 2 Statement 3 Incorrect Correct Correct Article 275 empowers the Parliament to make grants to the states which are in need of financial assistance and not to every state. Also, different sums may be fixed for different states. These sums are charged on the Consolidated Fund of India every year. The statutory grants are given to the states on the recommendation of the Finance Commission. Note:
Grants-in-Aid to the States:
Besides sharing of taxes between the Centre and the states, the Constitution provides for grants-in-aid to the states from the Central resources. There are two types of grants-in-aid, viz, statutory grants and discretionary grants:
Statutory Grants:
- Article 275 empowers the Parliament to make grants to the states which are in need of financial assistance and not to every state. Also, different sums may be fixed for different states.
- These sums are charged on the Consolidated Fund of India every year.
- Apart from this general provision, the Constitution also provides for specific grants for promoting the welfare of the scheduled tribes in a state or for raising the level of administration of the scheduled areas in a state including the State of Assam.
- The statutory grants under Article 275 (both general and specific) are given to the states on the recommendation of the Finance Commission.
Discretionary Grants:
- Article 282 empowers both the Centre and the states to make any grants for any public purpose, even if it is not within their respective legislative competence. Under this provision, the Centre makes grants to the states.
- These grants are also known as discretionary grants, the reason being that the Centre is under no obligation to give these grants and the matter lies within its discretion. These grants have a two-fold purpose: to help the state financially to fulfil plan targets; and to give some leverage to the Centre to influence and coordinate state action to effectuate the national plan.
Incorrect
Solution (b)
Statement 1 Statement 2 Statement 3 Incorrect Correct Correct Article 275 empowers the Parliament to make grants to the states which are in need of financial assistance and not to every state. Also, different sums may be fixed for different states. These sums are charged on the Consolidated Fund of India every year. The statutory grants are given to the states on the recommendation of the Finance Commission. Note:
Grants-in-Aid to the States:
Besides sharing of taxes between the Centre and the states, the Constitution provides for grants-in-aid to the states from the Central resources. There are two types of grants-in-aid, viz, statutory grants and discretionary grants:
Statutory Grants:
- Article 275 empowers the Parliament to make grants to the states which are in need of financial assistance and not to every state. Also, different sums may be fixed for different states.
- These sums are charged on the Consolidated Fund of India every year.
- Apart from this general provision, the Constitution also provides for specific grants for promoting the welfare of the scheduled tribes in a state or for raising the level of administration of the scheduled areas in a state including the State of Assam.
- The statutory grants under Article 275 (both general and specific) are given to the states on the recommendation of the Finance Commission.
Discretionary Grants:
- Article 282 empowers both the Centre and the states to make any grants for any public purpose, even if it is not within their respective legislative competence. Under this provision, the Centre makes grants to the states.
- These grants are also known as discretionary grants, the reason being that the Centre is under no obligation to give these grants and the matter lies within its discretion. These grants have a two-fold purpose: to help the state financially to fulfil plan targets; and to give some leverage to the Centre to influence and coordinate state action to effectuate the national plan.
-
Question 13 of 35
13. Question
Consider the following statements:
- The Central government can borrow outside India upon the security of the Consolidated Fund of India.
- The state government cannot borrow outside India upon the security of the Consolidated Fund of that state.
Which of the above statements is/are correct?
Correct
Solution (c)
Statement 1 Statement 2 Correct Correct Under the article 292, Government of India, can borrow outside India and by Article 112 any debt repayable by government of India is charged expenditure on Consolidated Fund of India. India is not a confederation of the state, wherein states can establish relationship with others countries autonomously. Under the Article 293, the state government extends to borrowing within the territory of India upon the security of the Consolidated Fund of state within such limits. Note:
ARTICLE 292 ARTICLE 293 Executive power of the union extends to borrowing and regulated by the parliament. E.g., FRBM Act.
Executive power of the states extends to borrowing and regulated by the state legislative. The central government can borrow from internal and external source. E.g., Market borrowing, International Organisation loans
The state legislative can borrow from only internal source. It may from time to time be fixed by the parliament by law and to the giving of guarantee within such limits. It may from time to time be fixed under any law made by parliament, make loans to any state shall not exceeded article 292, give guarantee in respect of loans raised by the state shall be charged on the Consolidated fund of India. — In some cases, state may not without consent of the government of India raise any loan if there is still outstanding any part of a loan made to the state by the central government. Incorrect
Solution (c)
Statement 1 Statement 2 Correct Correct Under the article 292, Government of India, can borrow outside India and by Article 112 any debt repayable by government of India is charged expenditure on Consolidated Fund of India. India is not a confederation of the state, wherein states can establish relationship with others countries autonomously. Under the Article 293, the state government extends to borrowing within the territory of India upon the security of the Consolidated Fund of state within such limits. Note:
ARTICLE 292 ARTICLE 293 Executive power of the union extends to borrowing and regulated by the parliament. E.g., FRBM Act.
Executive power of the states extends to borrowing and regulated by the state legislative. The central government can borrow from internal and external source. E.g., Market borrowing, International Organisation loans
The state legislative can borrow from only internal source. It may from time to time be fixed by the parliament by law and to the giving of guarantee within such limits. It may from time to time be fixed under any law made by parliament, make loans to any state shall not exceeded article 292, give guarantee in respect of loans raised by the state shall be charged on the Consolidated fund of India. — In some cases, state may not without consent of the government of India raise any loan if there is still outstanding any part of a loan made to the state by the central government. -
Question 14 of 35
14. Question
Consider the following statements regarding Type of Questions:
- A Starred Question is one to which a member desires a written answer from the Minister in the House.
- An Unstarred Question is one to which oral answer is desired by the member.
Which of the above statements is/are correct?
Correct
Solution (d)
Statement 1 Statement 2 Incorrect Incorrect A Starred Question is one to which a member desires an oral answer from the Minister in the House and is required to be distinguished by him/her with an asterisk. An Unstarred Question is one to which written answer is desired by the member and is deemed to be laid on the Table of the House by Minister. Thus, it is not called for oral answer in the House and no supplementary question can be asked thereon.
Note:
Types of Questions:
Members have a right to ask questions to elicit information on matters of public importance within the special cognizance of the Ministers concerned.
The questions are of four types:
- Starred Questions-
- A Starred Question is one to which a member desires an oral answer from the Minister in the House and is required to be distinguished by him/her with an asterisk.
- Answer to such a question may be followed by supplementary questions by members.
- Unstarred Questions-
- An Unstarred Question is one to which written answer is desired by the member and is deemed to be laid on the Table of the House by Minister.
- Thus, it is not called for oral answer in the House and no supplementary question can be asked thereon.
- Short Notice Questions-
- A member may give a notice of question on a matter of public importance and of urgent character for oral answer at a notice less than 10 days prescribed as the minimum period of notice for asking a question in ordinary course.
- Such a question is known as ‘Short Notice Question’.
- Questions to Private Members-
- A Question may also be addressed to a Private Member (Under Rule 40 of the Rules of Procedure and Conduct of Business in Lok Sabha), provided that the subject matter of the question relates to some Bill, Resolution or other matter connected with the business of the House for which that Member is responsible.
- The procedure in regard to such questions is same as that followed in the case of questions addressed to a Minister with such variations as the Speaker may consider necessary.
Incorrect
Solution (d)
Statement 1 Statement 2 Incorrect Incorrect A Starred Question is one to which a member desires an oral answer from the Minister in the House and is required to be distinguished by him/her with an asterisk. An Unstarred Question is one to which written answer is desired by the member and is deemed to be laid on the Table of the House by Minister. Thus, it is not called for oral answer in the House and no supplementary question can be asked thereon.
Note:
Types of Questions:
Members have a right to ask questions to elicit information on matters of public importance within the special cognizance of the Ministers concerned.
The questions are of four types:
- Starred Questions-
- A Starred Question is one to which a member desires an oral answer from the Minister in the House and is required to be distinguished by him/her with an asterisk.
- Answer to such a question may be followed by supplementary questions by members.
- Unstarred Questions-
- An Unstarred Question is one to which written answer is desired by the member and is deemed to be laid on the Table of the House by Minister.
- Thus, it is not called for oral answer in the House and no supplementary question can be asked thereon.
- Short Notice Questions-
- A member may give a notice of question on a matter of public importance and of urgent character for oral answer at a notice less than 10 days prescribed as the minimum period of notice for asking a question in ordinary course.
- Such a question is known as ‘Short Notice Question’.
- Questions to Private Members-
- A Question may also be addressed to a Private Member (Under Rule 40 of the Rules of Procedure and Conduct of Business in Lok Sabha), provided that the subject matter of the question relates to some Bill, Resolution or other matter connected with the business of the House for which that Member is responsible.
- The procedure in regard to such questions is same as that followed in the case of questions addressed to a Minister with such variations as the Speaker may consider necessary.
-
Question 15 of 35
15. Question
Joint sitting can be summoned for which of the following reasons?
- If the bill has been passed by one House and is rejected by the other House.
- If the Houses have finally disagreed as to the amendments to be made in the bill.
- If more than three months have elapsed from the date of the receipt of the bill by the other House without the bill being passed by it.
How many of the above statements are correct?
Correct
Solution (b)
JOINT SITTING OF TWO HOUSES:
- Joint sitting is an extraordinary machinery provided by the Constitution to resolve a deadlock between the two Houses over the passage of a bill.
- A deadlock is deemed to have taken place under any one of the following three situations after a bill has been passed by one House and transmitted to the other House:
- If the bill is rejected by the other House;
- If the Houses have finally disagreed as to the amendments to be made in the bill; or
- If more than six months have elapsed from the date of the receipt of the bill by the other House without the bill being passed by it.
- In the above three situations, the president can summon both the Houses to meet in a joint sitting for the purpose of deliberating and voting on the bill. It must be noted here that the provision of joint sitting is applicable to ordinary bills or financial bills only and not to money bills or Constitutional amendment bills.
- The Speaker of Lok Sabha presides over a joint sitting of the two Houses and the Deputy Speaker, in his absence. If the Deputy Speaker is also absent from a joint sitting, the Deputy Chairman of Rajya Sabha presides.
- The quorum to constitute a joint sitting is one-tenth of the total number of members of the two Houses. The joint sitting is governed by the Rules of Procedure of Lok Sabha and not of Rajya Sabha.
Incorrect
Solution (b)
JOINT SITTING OF TWO HOUSES:
- Joint sitting is an extraordinary machinery provided by the Constitution to resolve a deadlock between the two Houses over the passage of a bill.
- A deadlock is deemed to have taken place under any one of the following three situations after a bill has been passed by one House and transmitted to the other House:
- If the bill is rejected by the other House;
- If the Houses have finally disagreed as to the amendments to be made in the bill; or
- If more than six months have elapsed from the date of the receipt of the bill by the other House without the bill being passed by it.
- In the above three situations, the president can summon both the Houses to meet in a joint sitting for the purpose of deliberating and voting on the bill. It must be noted here that the provision of joint sitting is applicable to ordinary bills or financial bills only and not to money bills or Constitutional amendment bills.
- The Speaker of Lok Sabha presides over a joint sitting of the two Houses and the Deputy Speaker, in his absence. If the Deputy Speaker is also absent from a joint sitting, the Deputy Chairman of Rajya Sabha presides.
- The quorum to constitute a joint sitting is one-tenth of the total number of members of the two Houses. The joint sitting is governed by the Rules of Procedure of Lok Sabha and not of Rajya Sabha.
-
Question 16 of 35
16. Question
In the context of the enactment of the Budget, the Constitution of India contains which of the following provisions?
- No demand for grants shall be made except on the recommendation of the President
- No money shall be withdrawn from the Consolidated Fund of India except under appropriation made by law.
- No tax shall be levied or collected except by authority of law.
How many of the above statements are correct?
Correct
Solution (c)
Statement 1 Statement 2 Statement 3 Correct Correct Correct No demand for grants shall be made except on the recommendation of the President. Demands for grants are presented to the Parliament along with the budget.
These demands for grants show that the estimates of the expenditure for various departments and they need to be voted by the Parliament.
After the demands for grants are voted by the parliament, the Appropriation Bill is introduced, considered and passed by the appropriation of the Parliament. It provides the legal authority for withdrawal of funds of what is known as the Consolidated Fund of India.
No tax shall be levied or collected except by authority of law. Note:
Budget:
- The finance minister presents the budget in the Lok Sabha. He makes his budget in the Lok Sabha. Simultaneously, the copy of the budget is laid on the table of the Rajya Sabha. Printed copies of the budget are distributed among the members of the parliament to go through the details of the budgetary provisions.
- The finance bill is presented to the parliament immediately after the presentation of the budget. Finance Bill relates to the proposals regarding the imposition of new taxes, modification on the existing taxes or the abolition of the old taxes.
- The proposals on revenue and expenditure are discussed in the Parliament. Members of the Parliament actively take part in the discussion.
- After the general discussion on the budget is over, the Houses are adjourned for about three to four weeks. During this gap period, the 24 departmental standing committees of Parliament examine and discuss in detail the demands for grants of the concerned ministers and prepare reports on them. These reports are submitted to both the Houses of Parliament for consideration.
- Demands for grants are presented to the Parliament along with the budget. These demands for grants show that the estimates of the expenditure for various departments and they need to be voted by the Parliament. No demand for grants shall be made except on the recommendation of the President.
- After the demands for grants are voted by the parliament, the Appropriation Bill is introduced, considered and passed by the appropriation of the Parliament. It provides the legal authority for withdrawal of funds of what is known as the Consolidated Fund of India.
- After the passing of the appropriation bill, finance bill is discussed and passed. At this stage, the members of the parliament can suggest and make some amendments which the finance minister can approve or reject.
- Appropriation bill and Finance bill are sent to Rajya Sabha. The Rajya Sabha is required to send back these bills to the Lok Sabha within fourteen days with or without amendments. However, Lok Sabha may or may not accept the bill.
- Finance Bill is sent to the President for his assent. The bill becomes the statue after President’s sign. The president does not have the power to reject the bill.
Incorrect
Solution (c)
Statement 1 Statement 2 Statement 3 Correct Correct Correct No demand for grants shall be made except on the recommendation of the President. Demands for grants are presented to the Parliament along with the budget.
These demands for grants show that the estimates of the expenditure for various departments and they need to be voted by the Parliament.
After the demands for grants are voted by the parliament, the Appropriation Bill is introduced, considered and passed by the appropriation of the Parliament. It provides the legal authority for withdrawal of funds of what is known as the Consolidated Fund of India.
No tax shall be levied or collected except by authority of law. Note:
Budget:
- The finance minister presents the budget in the Lok Sabha. He makes his budget in the Lok Sabha. Simultaneously, the copy of the budget is laid on the table of the Rajya Sabha. Printed copies of the budget are distributed among the members of the parliament to go through the details of the budgetary provisions.
- The finance bill is presented to the parliament immediately after the presentation of the budget. Finance Bill relates to the proposals regarding the imposition of new taxes, modification on the existing taxes or the abolition of the old taxes.
- The proposals on revenue and expenditure are discussed in the Parliament. Members of the Parliament actively take part in the discussion.
- After the general discussion on the budget is over, the Houses are adjourned for about three to four weeks. During this gap period, the 24 departmental standing committees of Parliament examine and discuss in detail the demands for grants of the concerned ministers and prepare reports on them. These reports are submitted to both the Houses of Parliament for consideration.
- Demands for grants are presented to the Parliament along with the budget. These demands for grants show that the estimates of the expenditure for various departments and they need to be voted by the Parliament. No demand for grants shall be made except on the recommendation of the President.
- After the demands for grants are voted by the parliament, the Appropriation Bill is introduced, considered and passed by the appropriation of the Parliament. It provides the legal authority for withdrawal of funds of what is known as the Consolidated Fund of India.
- After the passing of the appropriation bill, finance bill is discussed and passed. At this stage, the members of the parliament can suggest and make some amendments which the finance minister can approve or reject.
- Appropriation bill and Finance bill are sent to Rajya Sabha. The Rajya Sabha is required to send back these bills to the Lok Sabha within fourteen days with or without amendments. However, Lok Sabha may or may not accept the bill.
- Finance Bill is sent to the President for his assent. The bill becomes the statue after President’s sign. The president does not have the power to reject the bill.
-
Question 17 of 35
17. Question
Consider the following statements:
- A Bill pending in the Council of States, which has not been passed by the House of the People, shall not lapse on the dissolution of the House of the People.
- A Bill pending in the Parliament shall not lapse by reason of the prorogation of the Houses.
- A Bill, which has been passed by the House of the People and is pending in the Council of States, shall not lapse on the dissolution of the House of the People.
How many of the above statements are correct?
Correct
Solution (b)
Statement 1 Statement 2 Statement 3 Correct Correct Incorrect A Bill pending in the Council of States, which has not been passed by the House of the People, shall not lapse on the dissolution of the House of the People. A Bill pending in the Parliament shall not lapse by reason of the prorogation of the Houses. A Bill, which has been passed by the House of the People and is pending in the Council of States, shall lapse on the dissolution of the House of the People. Notes:
- When the Lok Sabha is dissolved, all business including bills, motions, resolutions, notices, petitions and so on pending before it or its committees lapse.
- They (to be pursued further) must be reintroduced in the newly constituted Lok Sabha. However, some pending bills and all pending assurances that are to be examined by the Committee on Government Assurances do not lapse on the dissolution of the Lok Sabha.
- The position with respect to lapsing of bills is as follows:
- A bill pending in the Lok Sabha lapses (whether originating in the Lok Sabha or transmitted to it by the Rajya Sabha).
- A bill passed by the Lok Sabha but pending in the Rajya Sabha lapses.
- A bill not passed by the two Houses due to disagreement and if the president has notified the holding of a joint sitting before the dissolution of Lok Sabha, does not lapse.
- A bill pending in the Rajya Sabha but not passed by the Lok Sabha does not lapse.
- A bill passed by both Houses but pending assent of the president does not lapse.
- A bill passed by both Houses but returned by the president for reconsideration of Houses does not lapse.
Incorrect
Solution (b)
Statement 1 Statement 2 Statement 3 Correct Correct Incorrect A Bill pending in the Council of States, which has not been passed by the House of the People, shall not lapse on the dissolution of the House of the People. A Bill pending in the Parliament shall not lapse by reason of the prorogation of the Houses. A Bill, which has been passed by the House of the People and is pending in the Council of States, shall lapse on the dissolution of the House of the People. Notes:
- When the Lok Sabha is dissolved, all business including bills, motions, resolutions, notices, petitions and so on pending before it or its committees lapse.
- They (to be pursued further) must be reintroduced in the newly constituted Lok Sabha. However, some pending bills and all pending assurances that are to be examined by the Committee on Government Assurances do not lapse on the dissolution of the Lok Sabha.
- The position with respect to lapsing of bills is as follows:
- A bill pending in the Lok Sabha lapses (whether originating in the Lok Sabha or transmitted to it by the Rajya Sabha).
- A bill passed by the Lok Sabha but pending in the Rajya Sabha lapses.
- A bill not passed by the two Houses due to disagreement and if the president has notified the holding of a joint sitting before the dissolution of Lok Sabha, does not lapse.
- A bill pending in the Rajya Sabha but not passed by the Lok Sabha does not lapse.
- A bill passed by both Houses but pending assent of the president does not lapse.
- A bill passed by both Houses but returned by the president for reconsideration of Houses does not lapse.
-
Question 18 of 35
18. Question
A parliamentary committee means a committee that-
- Is appointed or elected by the House or nominated by the Speaker / Chairman
- Works under the direction of the Speaker / Chairman
- Presents its report to the House or to the Speaker / Chairman
- Has a secretariat provided by the Lok Sabha / Rajya Sabha
Select the correct answer using the code given below:
Correct
Solution (d)
Statement 1 Statement 2 Statement 3 Statement 4 Correct Correct Correct Correct Is appointed or elected by the House or nominated by the Speaker / Chairman Works under the direction of the Speaker / Chairman Presents its report to the House or to the Speaker / Chairman Has a secretariat provided by the Lok Sabha / Rajya Sabha Parliamentary Committees:
The Parliament is too unwieldy a body to deliberate effectively the issues that come up before it. The functions of the Parliament are varied, complex and voluminous. Moreover, it has neither the adequate time nor necessary expertise to make a detailed scrutiny of all legislative measures and other matters. Therefore, it is assisted by a number of committees in the discharge of its duties.
The Constitution of India makes a mention of these committees at different places, but without making any specific provisions regarding their composition, tenure, functions, etc. All these matters are dealt by the rules of two Houses.
Accordingly, a parliamentary committee means a committee that:
- Is appointed or elected by the House or nominated by the Speaker / Chairman
- Works under the direction of the Speaker / Chairman
- Presents its report to the House or to the Speaker / Chairman
- Has a secretariat provided by the Lok Sabha / Rajya Sabha
The consultative committees, which also consist of members of Parliament, are not parliamentary committees as they do not fulfill above four conditions.
Incorrect
Solution (d)
Statement 1 Statement 2 Statement 3 Statement 4 Correct Correct Correct Correct Is appointed or elected by the House or nominated by the Speaker / Chairman Works under the direction of the Speaker / Chairman Presents its report to the House or to the Speaker / Chairman Has a secretariat provided by the Lok Sabha / Rajya Sabha Parliamentary Committees:
The Parliament is too unwieldy a body to deliberate effectively the issues that come up before it. The functions of the Parliament are varied, complex and voluminous. Moreover, it has neither the adequate time nor necessary expertise to make a detailed scrutiny of all legislative measures and other matters. Therefore, it is assisted by a number of committees in the discharge of its duties.
The Constitution of India makes a mention of these committees at different places, but without making any specific provisions regarding their composition, tenure, functions, etc. All these matters are dealt by the rules of two Houses.
Accordingly, a parliamentary committee means a committee that:
- Is appointed or elected by the House or nominated by the Speaker / Chairman
- Works under the direction of the Speaker / Chairman
- Presents its report to the House or to the Speaker / Chairman
- Has a secretariat provided by the Lok Sabha / Rajya Sabha
The consultative committees, which also consist of members of Parliament, are not parliamentary committees as they do not fulfill above four conditions.
-
Question 19 of 35
19. Question
A bill is not to be deemed to be a money bill by reason only that it provides for-
Correct
Solution (b)
Money Bills:
Article 110 of the Constitution deals with the definition of money bills. It states that a bill is deemed to be a money bill if it contains ‘only’ provisions dealing with all or any of the following matters:
- The imposition, abolition, remission, alteration or regulation of any tax;
- The regulation of the borrowing of money by the Union government;
- The custody of the Consolidated Fund of India or the contingency fund of India, the payment of moneys into or the withdrawal of money from any such fund;
- The appropriation of money out of the Consolidated Fund of India;
- Declaration of any expenditure charged on the Consolidated Fund of India or increasing the amount of any such expenditure;
- The receipt of money on account of the Consolidated Fund of India or the public account of India or the custody or issue of such money, or the audit of the accounts of the Union or of a state; or
- Any matter incidental to any of the matters specified above.
However, a bill is not to be deemed to be a money bill by reason only that it provides for:
- The imposition of fines or other pecuniary penalties.
- The demand or payment of fees for licenses or fees for services rendered; or
- The imposition, abolition, remission, alteration or regulation of any tax by any local authority or body for local purposes.
Incorrect
Solution (b)
Money Bills:
Article 110 of the Constitution deals with the definition of money bills. It states that a bill is deemed to be a money bill if it contains ‘only’ provisions dealing with all or any of the following matters:
- The imposition, abolition, remission, alteration or regulation of any tax;
- The regulation of the borrowing of money by the Union government;
- The custody of the Consolidated Fund of India or the contingency fund of India, the payment of moneys into or the withdrawal of money from any such fund;
- The appropriation of money out of the Consolidated Fund of India;
- Declaration of any expenditure charged on the Consolidated Fund of India or increasing the amount of any such expenditure;
- The receipt of money on account of the Consolidated Fund of India or the public account of India or the custody or issue of such money, or the audit of the accounts of the Union or of a state; or
- Any matter incidental to any of the matters specified above.
However, a bill is not to be deemed to be a money bill by reason only that it provides for:
- The imposition of fines or other pecuniary penalties.
- The demand or payment of fees for licenses or fees for services rendered; or
- The imposition, abolition, remission, alteration or regulation of any tax by any local authority or body for local purposes.
-
Question 20 of 35
20. Question
The Speaker of Lok Sabha is the President of all the Parliamentary Forums except the:
Correct
Solution (a)
Parliamentary Forums:
- Parliamentary forums are not a closed group but provide a platform to the members to have interactions with the ministers concerned, experts and key officials from the nodal ministries with a view to have a focused and meaningful discussion on critical issues with a result-oriented approach for speeding up the implementation.
- The Speaker of Lok Sabha is the ex-officio President of all the Forums except the Parliamentary Forum on Population and Public Health wherein the Chairman of Rajya Sabha is the ex-officio President and the Speaker is the ex-officio Co-President.
- The Deputy Chairman of Rajya Sabha, the Deputy Speaker of Lok Sabha, the concerned Ministers and the Chairmen of Departmentally-Related Standing Committees are the ex-officio Vice-president of the respective Forums.
Incorrect
Solution (a)
Parliamentary Forums:
- Parliamentary forums are not a closed group but provide a platform to the members to have interactions with the ministers concerned, experts and key officials from the nodal ministries with a view to have a focused and meaningful discussion on critical issues with a result-oriented approach for speeding up the implementation.
- The Speaker of Lok Sabha is the ex-officio President of all the Forums except the Parliamentary Forum on Population and Public Health wherein the Chairman of Rajya Sabha is the ex-officio President and the Speaker is the ex-officio Co-President.
- The Deputy Chairman of Rajya Sabha, the Deputy Speaker of Lok Sabha, the concerned Ministers and the Chairmen of Departmentally-Related Standing Committees are the ex-officio Vice-president of the respective Forums.
-
Question 21 of 35
21. Question
Consider the following statements:
- JioSpaceFiber and Starlink use low Earth orbit (LEO) satellites.
- JioSpaceFiber is India’s first satellite-based gigabit internet service.
- JioSpaceFiber is currently only available in India while Starlink is available in over 30 countries.
How many of the above statements are correct?
Correct
Solution (b)
- JioSpaceFiber uses medium Earth orbit (MEO) satellites while Starlink uses low Earth orbit (LEO) satellites. Hence statement 1 is incorrect.
- MEO satellites orbiting at a greater distance from the planet will have higher latency (latency is the time that data takes to transfer across the network) than LEO satellites. However, MEO satellites can cover larger areas with fewer satellites.
- JioSpaceFiber is India’s first satellite-based gigabit internet service. Hence statement 2 is correct.
- It uses medium earth orbit (MEO) satellite technology to provide high-speed internet access to remote and underserved areas.
- Jio has partnered with SES to access the world’s latest medium earth orbit (MEO) satellite technology.
- JioSpaceFiber is currently only available in India while Starlink is available in over 30 countries. Hence statement 3 is correct.
Incorrect
Solution (b)
- JioSpaceFiber uses medium Earth orbit (MEO) satellites while Starlink uses low Earth orbit (LEO) satellites. Hence statement 1 is incorrect.
- MEO satellites orbiting at a greater distance from the planet will have higher latency (latency is the time that data takes to transfer across the network) than LEO satellites. However, MEO satellites can cover larger areas with fewer satellites.
- JioSpaceFiber is India’s first satellite-based gigabit internet service. Hence statement 2 is correct.
- It uses medium earth orbit (MEO) satellite technology to provide high-speed internet access to remote and underserved areas.
- Jio has partnered with SES to access the world’s latest medium earth orbit (MEO) satellite technology.
- JioSpaceFiber is currently only available in India while Starlink is available in over 30 countries. Hence statement 3 is correct.
-
Question 22 of 35
22. Question
Consider the following statements about the Right to Information (Amendment) Act of 2019:
- It provided that the salary, allowances, and other service conditions of the Chief Information Commissioner and an Information Commissioner shall be such as prescribed by the Central Government.
- It fixed the term of office of the Chief Information Commissioner and an Information Commissioner for five years.
Which of the statements given above is/are correct?
Correct
Solution (a)
The Right to Information Act 2005 mandates timely response to citizen requests for government information.
The Right to Information (Amendment) Act of 2019:
- It provided that the salary, allowances, and other service conditions of the Chief Information Commissioner and an Information Commissioner shall be such as prescribed by the Central Government. Hence statement 1 is correct.
- Before this amendment, the salary, allowances, and other service conditions of the Chief Information Commissioner were similar to those of the Chief Election Commissioner and that of an Information Commissioner were similar to those of an Election Commissioner.
- It provided that the Chief Information Commissioner and an Information Commissioner shall hold office for such term as prescribed by the Central Government. Before this amendment, their term was fixed for 5 years. Hence statement 2 is incorrect.
- It removed the provisions regarding deductions in the salary of the Chief Information Commissioner, an Information Commissioner, the State Chief Information Commissioner, and a State Information Commissioner due to pension or any other retirement benefits received by them for their previous government service.
Incorrect
Solution (a)
The Right to Information Act 2005 mandates timely response to citizen requests for government information.
The Right to Information (Amendment) Act of 2019:
- It provided that the salary, allowances, and other service conditions of the Chief Information Commissioner and an Information Commissioner shall be such as prescribed by the Central Government. Hence statement 1 is correct.
- Before this amendment, the salary, allowances, and other service conditions of the Chief Information Commissioner were similar to those of the Chief Election Commissioner and that of an Information Commissioner were similar to those of an Election Commissioner.
- It provided that the Chief Information Commissioner and an Information Commissioner shall hold office for such term as prescribed by the Central Government. Before this amendment, their term was fixed for 5 years. Hence statement 2 is incorrect.
- It removed the provisions regarding deductions in the salary of the Chief Information Commissioner, an Information Commissioner, the State Chief Information Commissioner, and a State Information Commissioner due to pension or any other retirement benefits received by them for their previous government service.
-
Question 23 of 35
23. Question
Consider the following statements about Baba Farid:
- He followed the Chishti Order and was among the first Sufi saints to compose verses in Punjabi.
- Many of his verses are enshrined in the Guru Granth Sahib, the holy scripture of Sikhs.
Which of the statements given above is/are correct?
Correct
Solution (c)
- Baba Farid, born in 1173 CE near Shakarganj, near Multan (Pakistan), belonged to a family that had migrated from Kabul to Punjab.
- Baba Farid followed the Chishti Orderand was among the first Sufi saints to compose verses in Punjabi. He emphasized love, devotion, and unity with God. Hence statement 1 is correct.
- Many of his verses are enshrined in the Guru Granth Sahib, the holy scripture of Sikhs. Hence statement 2 is correct.
- Baba Farid is said to have meditated at a place in Jerusalem for 40 days. Baba Farid during his travels to Jerusalem prayed at the Al-Aqsa Mosque and stayed at a lodge. This lodge later came to be known as Zawiya Al-Hindiya, meaning “the Lodge of Hind.” After Baba Farid’s departure, the lodge maintained its association with India.
- The medieval traveller Evliya Chelebi described Zawiya Al-Hindiya as one of the largest Zawiyas in the city in 1671. This lodge is currently under the ownership of the Waqf Board of India and is accessible only to individuals of Indian citizenship or heritage.
Incorrect
Solution (c)
- Baba Farid, born in 1173 CE near Shakarganj, near Multan (Pakistan), belonged to a family that had migrated from Kabul to Punjab.
- Baba Farid followed the Chishti Orderand was among the first Sufi saints to compose verses in Punjabi. He emphasized love, devotion, and unity with God. Hence statement 1 is correct.
- Many of his verses are enshrined in the Guru Granth Sahib, the holy scripture of Sikhs. Hence statement 2 is correct.
- Baba Farid is said to have meditated at a place in Jerusalem for 40 days. Baba Farid during his travels to Jerusalem prayed at the Al-Aqsa Mosque and stayed at a lodge. This lodge later came to be known as Zawiya Al-Hindiya, meaning “the Lodge of Hind.” After Baba Farid’s departure, the lodge maintained its association with India.
- The medieval traveller Evliya Chelebi described Zawiya Al-Hindiya as one of the largest Zawiyas in the city in 1671. This lodge is currently under the ownership of the Waqf Board of India and is accessible only to individuals of Indian citizenship or heritage.
-
Question 24 of 35
24. Question
Consider the following statements about the activities under the “Meri Maati Mera Desh Campaign”:
- Veeron Ka Vandan will felicitate freedom fighters and the families of deceased freedom fighters.
- Vasudha Vandhan will encourage every gram panchayat to plant 75 saplings of indigenous species.
- Shilaphalakams will be built in every village or panchayat to pay tribute to those who laid down their lives during the freedom struggle.
How many of the above statements are correct?
Correct
Solution (c)
Meri Maati Mera Desh Campaign aims to honour the brave freedom fighters and brave hearts who sacrificed their lives for the country. The Ministry of Culture is the nodal ministry for this scheme.
The activities under the Meri Maati Mera Desh Campaign:
- Veeron Ka Vandan will felicitate freedom fighters and the families of deceased freedom fighters. Hence statement 1 is correct.
- Vasudha Vandhan will encourage every gram panchayat to plant 75 saplings of indigenous species. Hence statement 2 is correct.
- Shilaphalakams will be built in every village or panchayat to pay tribute to those who laid down their lives during the freedom struggle. Hence statement 3 is correct.
- Taking the Panch Pran Pledge with a focus on making India a developed country, eliminating the mentality of slavery, being proud of our rich heritage, upholding unity and solidarity, fulfilling duties as citizens, and respecting those who protect the nation.
- Volunteers have been tasked with collecting “mitti” from each of the panchayats or villages and bringing them to the block level. From each block, a “mitti kalash”, will be carried to Delhi. The soil will be utilized to develop a unique garden, called Amrit Vatika.
Incorrect
Solution (c)
Meri Maati Mera Desh Campaign aims to honour the brave freedom fighters and brave hearts who sacrificed their lives for the country. The Ministry of Culture is the nodal ministry for this scheme.
The activities under the Meri Maati Mera Desh Campaign:
- Veeron Ka Vandan will felicitate freedom fighters and the families of deceased freedom fighters. Hence statement 1 is correct.
- Vasudha Vandhan will encourage every gram panchayat to plant 75 saplings of indigenous species. Hence statement 2 is correct.
- Shilaphalakams will be built in every village or panchayat to pay tribute to those who laid down their lives during the freedom struggle. Hence statement 3 is correct.
- Taking the Panch Pran Pledge with a focus on making India a developed country, eliminating the mentality of slavery, being proud of our rich heritage, upholding unity and solidarity, fulfilling duties as citizens, and respecting those who protect the nation.
- Volunteers have been tasked with collecting “mitti” from each of the panchayats or villages and bringing them to the block level. From each block, a “mitti kalash”, will be carried to Delhi. The soil will be utilized to develop a unique garden, called Amrit Vatika.
-
Question 25 of 35
25. Question
Consider the following statements about Venezuela:
- It is bordered by Colombia, Brazil, Guyana, and the Caribbean Sea.
- It has the world’s largest oil reserves.
- It is located in Northern part of South America with Caracas as its capital.
How many of the above statements are correct?
Correct
Solution (c)
- Venezuela is bordered by Colombia, Brazil, Guyana, and the Caribbean Sea. Hence statement 1 is correct.
- It has the world’s largest oil reserves. Hence statement 2 is correct.
- It relies heavily on oil exports.
- It is located in Northern part of South America with Caracas as its capital. Hence statement 3 is correct.
Image source: WorldAtlas
Incorrect
Solution (c)
- Venezuela is bordered by Colombia, Brazil, Guyana, and the Caribbean Sea. Hence statement 1 is correct.
- It has the world’s largest oil reserves. Hence statement 2 is correct.
- It relies heavily on oil exports.
- It is located in Northern part of South America with Caracas as its capital. Hence statement 3 is correct.
Image source: WorldAtlas
-
Question 26 of 35
26. Question
Consider the following statements about the NexCAR19:
- It is India’s first indigenously developed CAR-T cell therapy.
- It is designed to target cancer cells carrying the CD19 protein.
Which of the statements given above is/are correct?
Correct
Solution (c)
- The NexCAR19 is India’s first indigenously developed CAR-T cell therapy. Hence statement 1 is correct.
- India is one of the first developing countries with its indigenous CAR-T and gene therapy platform.
- It was developed by ImmunoACT, a company incubated by IIT Bombay.
- It is recommended for people with B-cell lymphomas who didn’t respond to standard treatments like chemotherapy, leading to relapse or recurrence of the cancer.
- It is designed to target cancer cells carrying the CD19 protein. Hence statement 2 is correct.
Incorrect
Solution (c)
- The NexCAR19 is India’s first indigenously developed CAR-T cell therapy. Hence statement 1 is correct.
- India is one of the first developing countries with its indigenous CAR-T and gene therapy platform.
- It was developed by ImmunoACT, a company incubated by IIT Bombay.
- It is recommended for people with B-cell lymphomas who didn’t respond to standard treatments like chemotherapy, leading to relapse or recurrence of the cancer.
- It is designed to target cancer cells carrying the CD19 protein. Hence statement 2 is correct.
-
Question 27 of 35
27. Question
Which of the following Indian cities are part of the UNESCO Creative Cities Network?
- Kozhikode
- Bengaluru
- Chennai
- Srinagar
Choose the correct code:
Correct
Solution (c)
The UNESCO Creative Cities Network was launched in 2004 and aims to promote cooperation among cities which recognized creativity as a major factor in their urban development. Recently, Gwalior and Kozhikode from India joined the 55 new cities which have been included in the UNESCO Creative Cities Network.
There are seven categories– Design, Film, Gastronomy, Literature, Media Arts, Music, and Crafts & Folk Art. With the latest additions, the UCCN now counts 350 cities in more than a hundred countries
Do You Know?: World Cities Day designated by the United Nations on October 31.
The newly designated Creative Cities are invited to participate in the 2024 UCCN Annual Conference (July 1-5, 2024) in Braga, Portugal, under the theme “Bringing Youth to the table for the next decade”
Indian cities that are part of the UNESCO Creative Cities Network are:
Cities Category Kozhikode City of Literature Gwalior City of Music Srinagar Crafts and Folk Arts Mumbai Film Hyderabad Gastronomy Chennai City of Music Jaipur Crafts and Folk Arts Varanasi City of Music Hence option c is correct.
Incorrect
Solution (c)
The UNESCO Creative Cities Network was launched in 2004 and aims to promote cooperation among cities which recognized creativity as a major factor in their urban development. Recently, Gwalior and Kozhikode from India joined the 55 new cities which have been included in the UNESCO Creative Cities Network.
There are seven categories– Design, Film, Gastronomy, Literature, Media Arts, Music, and Crafts & Folk Art. With the latest additions, the UCCN now counts 350 cities in more than a hundred countries
Do You Know?: World Cities Day designated by the United Nations on October 31.
The newly designated Creative Cities are invited to participate in the 2024 UCCN Annual Conference (July 1-5, 2024) in Braga, Portugal, under the theme “Bringing Youth to the table for the next decade”
Indian cities that are part of the UNESCO Creative Cities Network are:
Cities Category Kozhikode City of Literature Gwalior City of Music Srinagar Crafts and Folk Arts Mumbai Film Hyderabad Gastronomy Chennai City of Music Jaipur Crafts and Folk Arts Varanasi City of Music Hence option c is correct.
-
Question 28 of 35
28. Question
Recently in the news, the Bletchley Declaration is related to?
Correct
Solution (d)
The Bletchley Declaration is related to Artificial Intelligence. It addresses risks and responsibilities associated with frontier AI comprehensively and collaboratively. Its member countries include Australia, Brazil, Canada, Chile, China, France, Germany, India, Indonesia, Ireland, Israel, Italy, Japan, Kenya, Saudi, Arabia, Netherlands, Nigeria, The Philippines, the Republic of Korea, Rwanda, Singapore, Spain, Switzerland, Turkey, Ukraine, United Arab Emirates, United Kingdom of Great Britain and Northern Ireland, the United States of America, and the European Union. Hence option d is correct.
Incorrect
Solution (d)
The Bletchley Declaration is related to Artificial Intelligence. It addresses risks and responsibilities associated with frontier AI comprehensively and collaboratively. Its member countries include Australia, Brazil, Canada, Chile, China, France, Germany, India, Indonesia, Ireland, Israel, Italy, Japan, Kenya, Saudi, Arabia, Netherlands, Nigeria, The Philippines, the Republic of Korea, Rwanda, Singapore, Spain, Switzerland, Turkey, Ukraine, United Arab Emirates, United Kingdom of Great Britain and Northern Ireland, the United States of America, and the European Union. Hence option d is correct.
-
Question 29 of 35
29. Question
Consider the following border crossings:
- Kerem Shalom – Between Gaza and Israel.
- Rafah – Between Gaza Strip and Egypt.
- Erez – Between Gaza and Jordan.
How many of the given above are correctly matched?
Correct
Solution (b)
Border crossings:
- Kerem Shalom – Between Southern Gaza and Israel.
- Rafah – Between Gaza Strip and Egypt.
- Erez – Between Northern Gaza and Israel.
Hence option b is correct.
Image Source: Indian Express
Incorrect
Solution (b)
Border crossings:
- Kerem Shalom – Between Southern Gaza and Israel.
- Rafah – Between Gaza Strip and Egypt.
- Erez – Between Northern Gaza and Israel.
Hence option b is correct.
Image Source: Indian Express
-
Question 30 of 35
30. Question
Which of the following are the applications of carbon nanoflorets?
- It can remove up to 90% of pollutants containing arsenic, chromium, cadmium, and mercury.
- It can generate heat sustainably, without having to burn fossil fuels.
- Its coatings can help heat housing and sterilise surfaces in hospitals.
Which of the statements given above is/are correct?
Correct
Solution (c)
Carbon nanoflorets are a unique nanostructure composed of carbon atoms arranged in a distinctive floret-like morphology. They have a high surface area and various potential applications in fields such as materials science, electronics, and nanotechnology due to their exceptional properties and structure.
The applications of carbon nanoflorets:
- It can remove up to 90% of pollutants containing arsenic, chromium, cadmium, and mercury. Hence statement 1 is correct.
- It can generate heat sustainably, without having to burn fossil fuels. Hence statement 2 is correct.
- Its coatings can help heat housing and sterilise surfaces in hospitals. Hence statement 3 is correct.
- It can absorb light at multiple frequencies like infrared, visible, and ultraviolet.
Incorrect
Solution (c)
Carbon nanoflorets are a unique nanostructure composed of carbon atoms arranged in a distinctive floret-like morphology. They have a high surface area and various potential applications in fields such as materials science, electronics, and nanotechnology due to their exceptional properties and structure.
The applications of carbon nanoflorets:
- It can remove up to 90% of pollutants containing arsenic, chromium, cadmium, and mercury. Hence statement 1 is correct.
- It can generate heat sustainably, without having to burn fossil fuels. Hence statement 2 is correct.
- Its coatings can help heat housing and sterilise surfaces in hospitals. Hence statement 3 is correct.
- It can absorb light at multiple frequencies like infrared, visible, and ultraviolet.
-
Question 31 of 35
31. Question
In a row of Children facing north, K is 15th to the left of H, who is 22nd from the right end. If P is 14th from the left end and 6th to the right of K, how many children are there in that row?
Correct
Solution (d)
Here, P is 14th from the left end and 6th to the right of K. Hence, K is 8th from the left side.
Again, K is 15thto the left of H, who is 22nd from the right end.
Hence, K is 37th from the right side.
Therefore, the total number of Children is
= K’s position from the right ends + K’s position from the left ends – 1
= 37 + 8 – 1 = 45 – 1 = 44
Incorrect
Solution (d)
Here, P is 14th from the left end and 6th to the right of K. Hence, K is 8th from the left side.
Again, K is 15thto the left of H, who is 22nd from the right end.
Hence, K is 37th from the right side.
Therefore, the total number of Children is
= K’s position from the right ends + K’s position from the left ends – 1
= 37 + 8 – 1 = 45 – 1 = 44
-
Question 32 of 35
32. Question
The digit at unit place of the number (1640)^2 + (1641)^2 + (1662)^3 + (1693)^2 is:
Correct
Solution (c)
(1640)^2 = unit digit is 0
(1641)^2 = unit digit is 1
(1662)^3 = unit digit is 2^3 = 8
(1693)^2 = unit digit is 3^2 = 9
So, 0 +1+ 8 + 9 = 18, unit digit is 8
Incorrect
Solution (c)
(1640)^2 = unit digit is 0
(1641)^2 = unit digit is 1
(1662)^3 = unit digit is 2^3 = 8
(1693)^2 = unit digit is 3^2 = 9
So, 0 +1+ 8 + 9 = 18, unit digit is 8
-
Question 33 of 35
33. Question
XYZ is a 3 digit number such that when we calculate the difference between the two three-digit numbers XYZ-YXZ the difference is exactly 90. How many possible values exist for the digits X and Y?
Correct
Solution (a)
First of all we get that the digits X and Y cannot accept the value of zero because both of them are occurring at the hundredth’s place.
Now expanding both the numbers in terms ones, tens, and hundreds we get,
XYZ = 100X + 10Y + Z, YXZ = 100Y + 10X + Z
Now we are given the difference of both numbers,
XYZ-YXZ = 90
100X + 10Y + Z – 100Y + 10X + Z = 90
90X – 90Y = 90
X – Y = 1
Therefore we can say that the values of X and Y are in consecutive order of numbers starting from one.
The possible values of X and Y are,
(X,Y) = (2,1), (3,2), (4,3), (5,4), (6,5), (7,6), (8,7), (9,8)
Hence, there are existing eight possible values of both X and Y for the required scenario.
Incorrect
Solution (a)
First of all we get that the digits X and Y cannot accept the value of zero because both of them are occurring at the hundredth’s place.
Now expanding both the numbers in terms ones, tens, and hundreds we get,
XYZ = 100X + 10Y + Z, YXZ = 100Y + 10X + Z
Now we are given the difference of both numbers,
XYZ-YXZ = 90
100X + 10Y + Z – 100Y + 10X + Z = 90
90X – 90Y = 90
X – Y = 1
Therefore we can say that the values of X and Y are in consecutive order of numbers starting from one.
The possible values of X and Y are,
(X,Y) = (2,1), (3,2), (4,3), (5,4), (6,5), (7,6), (8,7), (9,8)
Hence, there are existing eight possible values of both X and Y for the required scenario.
-
Question 34 of 35
34. Question
Two numbers x and y are chosen at random from the set of first 30 natural numbers. The probability that x^2 – y^2 is divisible by 3 is
Correct
Solution (c)
Total number of ways of choosing 2 numbers from the set = 30 C2 = 30×29 /2=435
Now, x^2 –y^2 =(x+y)(x−y)
Two cases arise.
Case 1: When both x,y are divisible by 3
The numbers which are divisible by 3 = 3,6,9,….,30
Number of ways 2 numbers can be chosen from this set of numbers = 10 C2 = 10×9 /2 =45
Case 2: When both x,y are not divisible by 3
The numbers not divisible by 3 = (All the numbers in the set) – ( All the numbers divisible by 3) Hence, there are 20 numbers which aren’t divisible by 3.
Number of ways 2 numbers can be chosen from this set = 20 C 2 = (20×19)/2 =190
Thus, total number of favorable outcomes = 45 + 190 = 235
Thus, the required probability = 235/435 = 47 /87
Incorrect
Solution (c)
Total number of ways of choosing 2 numbers from the set = 30 C2 = 30×29 /2=435
Now, x^2 –y^2 =(x+y)(x−y)
Two cases arise.
Case 1: When both x,y are divisible by 3
The numbers which are divisible by 3 = 3,6,9,….,30
Number of ways 2 numbers can be chosen from this set of numbers = 10 C2 = 10×9 /2 =45
Case 2: When both x,y are not divisible by 3
The numbers not divisible by 3 = (All the numbers in the set) – ( All the numbers divisible by 3) Hence, there are 20 numbers which aren’t divisible by 3.
Number of ways 2 numbers can be chosen from this set = 20 C 2 = (20×19)/2 =190
Thus, total number of favorable outcomes = 45 + 190 = 235
Thus, the required probability = 235/435 = 47 /87
-
Question 35 of 35
35. Question
A, B and C can do a piece of work in 20,30 and 40 days respectively. A is assisted by B on one day and by C on the next day, alternatively. How long should the work take to complete?
Correct
Solution (b)
Work done by A, B and C respectively in 20, 30 and 40 days.
Total unit of work is equal to L. C. M. of 20, 30 and 40 = 120.
Efficiency of A 120/20 = 6 Units in 1 day
Efficiency of B 120/30 = 4 units in 1 day
Efficiency of C 120/40 = 3 units in 1 day
Work done by (A+B) in 1 day = 6 + 4 = 10 units
Work done by (A+C) in 1 day = 6 + 3 = 9 units
And this cycle continuous alternatively so, work done in 2 days = 10 + 9 = 19 units
Work done in 12 days = 114 units
Remaining work = 120 – 114 = 6 units
And this remaining work is done by (A +B) in 6/10 days
So, total work done = 12 + (6/10) = 12(3/5) days
Incorrect
Solution (b)
Work done by A, B and C respectively in 20, 30 and 40 days.
Total unit of work is equal to L. C. M. of 20, 30 and 40 = 120.
Efficiency of A 120/20 = 6 Units in 1 day
Efficiency of B 120/30 = 4 units in 1 day
Efficiency of C 120/40 = 3 units in 1 day
Work done by (A+B) in 1 day = 6 + 4 = 10 units
Work done by (A+C) in 1 day = 6 + 3 = 9 units
And this cycle continuous alternatively so, work done in 2 days = 10 + 9 = 19 units
Work done in 12 days = 114 units
Remaining work = 120 – 114 = 6 units
And this remaining work is done by (A +B) in 6/10 days
So, total work done = 12 + (6/10) = 12(3/5) days
All the Best
IASbaba